អ្នកប្រើប្រាស់:ថៃ ទេពី/sandbox៖ ភាពខុសគ្នារវាងកំណែនានា

ខ្លឹមសារដែលបានលុបចោល ខ្លឹមសារដែលបានសរសេរបន្ថែម
No edit summary
No edit summary
 
បន្ទាត់ទី១៖
<noinclude>
</noinclude>
{{User sandbox}}
<!-- EDIT BELOW THIS LINE -->
នៅក្នុង [[mathematics|គណិតវិទ្យា]],''' វិសមភាព​មធ្យមនព្វន្ត និង មធ្យមធរណីមាត្រ​''' ឬត្រូវបានហៅកាត់ថា''' វិសមភាព AM-GM''', បាននិយាយថា មធ្យមនព្វន្តនៃចំនួនពិតមិនអវិជ្ជមានមួយក្រុម គឺ​តែងតែធំជាងឬស្មើទៅនឹង មធ្យមធរណីមាត្រ នៃចំនួនពិតមិនអវិជ្ជមានមួយក្រុមនោះជានិច្ច។ ជាងនេះទៅទៀតនោះ, មធ្យមទាំងពីរនេះអាចស្មើគ្នាទៅបានលុះត្រាតែ គ្រប់ធាតុនីមួយៗ (ចំនួន ពិតមិនអវិជ្ចមាននីមួយៗ)នៅក្នុងក្រុមចំនួននោះ មានតម្លៃស្មើគ្នាទាំងអស់។
 
ក្នុងឧទាហរណ៍សាមញ្ញតែមានសារៈសំខាន់នោះគឺ —ដូចជាក្នុងករណីមានច្រើនជាង ១អញ្ញតិ — ចំពោះចំនួនពិតមិនអវិជ្ជមានពីរគឺ {{mvar|x}} និង&nbsp;{{mvar|y}}, ដែលវិសមភាពខាងក្រោមនេះពិតជានិច្ច៖
:<math>\frac{x+y}2 \ge \sqrt{xy}</math>
ដែលស្មើគ្នាលុះត្រាតែ {{math|''x'' {{=}} ''y''}} ប៉ុណ្នោះ. ក្នុងករណីនេះយើងអាចឃើញយ៉ាងច្បាស់ពីការពិតដែលការ៉េនៃចំនួនពិតមួយតែងតែវិជ្ជមានជានិច្ច គួបផ្សំនឹងលក្ខណៈ ដ៏ពិតមួយទៀងនោះគឺ {{math|(''a'' ± ''b'')<sup>2</sup> {{=}} ''a''<sup>2</sup> ± 2''ab'' + ''b''<sup>2</sup>}} ដែលជាលក្ខណៈនៃ [[binomial formula|រូបមន្តពហុធា]]:
: <math>0 = (x-y)^2</math>
: <math>0 = x^2-2xy+y^2 </math>
: <math>0 = x^2+2xy+y^2 - 4xy </math>
: <math>0 = (x+y)^2 - 4xy.</math>
ឬក៏អាចសរសេរជា<span style="line-height:1.5"> {{math|(''x'' + ''y'')<sup>2</sup> ≥ 4''xy''}}, ដែលស្មើគ្នានៅពេល {{math|(''x'' − ''y'')<sup>2</sup> {{=}} 0}} ឬពេល {{math|''x'' {{=}} ''y''}}. ចំពោះការពន្យល់តាមបែបធរណីមាត្រវិញ, ឧបមាថាយើងមាន [[rectangle|ត្រីកោណមួយ]] ដែលមានជ្រុងប្រវែង&nbsp;{{mvar|x}} និង&nbsp;{{mvar|y}}, នោះ [[perimeter|បរិមាត្រ]] របស់វាគឺ {{math|2''x'' + 2''y''}} និង [[area|ក្រលាផ្ទៃ]]&nbsp;{{mvar|xy}}. ដូចគ្នានេះដែរ, [[square|ការ៉េ]] មួយដែលគ្រប់ជ្រុងរបស់វាសុទ្ឋតែមានប្រវែង {{math|{{radical|''xy''}}}} មានបរិមាត្រស្មើនឹង {{math|4{{radical|''xy''}}}} និងមានក្រលាផ្ទៃដូចគ្នាទៅនឹងក្រលាផ្ទៃរបស់ត្រីកោណដែរ។ ករណីសាមញ្ញនៃវិសមភាព AM–GM ត្រូវបានអនុវត្តន៍លើបរិមាត្រគឺ {{math|2''x'' + 2''y'' ≥ 4{{radical|''xy''}}}} និងបានបង្ហាញថាមានតែការ៉េនោះទេដែលមានបរិមាត្រតូចជាងគេក្នុងចំនោមត្រីកោណទាំងអស់ដែលមានក្រលាផ្ទៃប៉ុនគ្នានោះ។</span>
 
ភាពទូទៅនៃវិសមភាព AM–GM ផ្ដោតសំខាន់ទៅលើភាពពិតនៃ [[natural logarithm|លោការីត ធម្មជាតិ]], ដែលអាចបម្លែងផលគុណទៅជាផលបូកបាននោះ, គឺជា is a [[concave function|អនុគមន៍ផតដាច់ខាត concave function]]​មួយ ហើយយើងអាចប្រើ [[Jensen's inequality|វិសមភាព Jensen]] ដើម្បី [[#Weighted_AM.E2.80.93GM_inequality|ស្រាយបញ្ជាក់ជាទូទៅ]] នៃវិសមភាព AM-GM នេះផងដែរ។
 
Extensions of the AM–GM inequality are available to include [[#Weighted_AM.E2.80.93GM_inequality|weights]] or [[generalized mean]]s.
 
== ប្រវត្តិ ឬ ព្រឹត្តិការណ៍ទាក់ទង ==
 
''មធ្យមពិជគណិត ''ឬនិយាយសាមញ្ញថា មធ្យម​ នៃ {{mvar|n}} ចំនួន {{math|''x''<sub>1</sub>, ''x''<sub>2</sub>, . . . , ''x<sub>n</sub>''}} គឺជាផលបូកនៃចំនួនទាំងអស់នោះ ចែកនឹង {{mvar|n}}។
 
:<math>\frac{x_1 + x_2 + \cdots + x_n}{n}.</math>
 
''មធ្យមធរណីមាត្រ'' មានភាពស្រដៀងគ្នា, ប៉ុន្តែវាមានន័យលុះត្រាតែចំនួនទាំងអស់គឺជា ចំនួនពិតមិនអវិជ្ជមានតែប៉ុណ្ណោះ ហើយប្រើប្រាស់នូវ ការគុណ និង [[Nth root|ឬស]] ចំនួសឲ្យ ការបូក និង ការចែក។
 
:<math>\sqrt[n]{x_1 \cdot x_2 \cdots x_n}.</math>
 
ករណី {{math|''x''<sub>1</sub>, ''x''<sub>2</sub>, . . . , ''x<sub>n</sub>'' > 0}},វាស្មើទៅនឹង [[exponential function|អ៊ិចស្ប៉ូណង់ស្យែល]] នៃមធ្យមពិជគណិតរបស់​ [[natural logarithm|លោការីត ធម្មជាតិ]] នៃចំនួនទាំងនោះ:
 
:<math>\exp \left( \frac{\ln {x_1} + \ln {x_2} + \cdots + \ln {x_n}}{n} \right).</math>
 
== វិសមភាព AM-GM ==
 
បញ្ជាក់សារជាថ្មីដោយប្រើប្រាស់សញ្ញាក្នុងគណិតវិទ្យា, ចំពោះ {{mvar|n}} ចំនួននៃចំនួនពិតមិនអវិជ្ជមាន {{math|''x''<sub>1</sub>, ''x''<sub>2</sub>, . . . , ''x<sub>n</sub>''}}, យើងបានៈ
 
:<math>\frac{x_1 + x_2 + \cdots + x_n}{n} \ge \sqrt[n]{x_1 \cdot x_2 \cdots x_n}\,,</math>
 
ហើយវិសមភាពនេះក្លាយជា សមភាព លុះត្រាតែៈ {{math|''x''<sub>1</sub> {{=}} ''x''<sub>2</sub> {{=}} · · · {{=}} ''x<sub>n</sub>''}}.
 
== ការពន្យល់តាមបែបធរណីមាត្រ ==
 
នៅក្នុង 2 វិមាត្រ , {{math|2''x''<sub>1</sub> + 2''x''<sub>2</sub>}} ជា[[perimeter|បរិមាត្រ]] នៃចតុកោណកែងមួយដែលមានជ្រុងប្រវែង {{math|''x''<sub>1</sub>}} និង&nbsp;{{math|''x''<sub>2</sub>}}។ ដួចគ្នានេះដែរ {{math|4{{radical|''x''<sub>1</sub>''x''<sub>2</sub>}}}} គឺជាបរិមាត្រនៃការ៉េមួយដែលមាន [[area|ក្រលាផ្ទៃ]] ដូចគ្នា. ដូចនេះ ចំពោះ {{math|''n'' {{=}} 2}} វិសមភាព AM–GM បានបញ្ជាក់ថា មានតែ ការ៉ប៉ុណ្ណោះដែលមានបរិមាត្រតូចជាងគេទាំងអស់ ក្នុងចំនោមចតុកោណកែងដែលមានក្រលាផ្ទៃប៉ុនគ្នា។
 
ភាពពេញលេញនៃវិសមភាពនេះ គឺជាការពង្រីកទៅលើគំនិតនេះ ទៅដល់ {{mvar|n}} វិមាត្រ។ គ្រប់កំពូលទាំងអស់នៃប្រអប់ដែលមាន {{mvar|n}}​ វិមាត្រគឺត្រូវបានភ្ជាប់ទៅនឹង {{mvar|n}} គែម។ ប្រសិនបើប្រវែងនៃគែមទាំងនោះគឺ​{{math|''x''<sub>1</sub>, ''x''<sub>2</sub>, . . . , ''x<sub>n</sub>''}}នោះ {{math|''x''<sub>1</sub> + ''x''<sub>2</sub> + · · · + ''x<sub>n</sub>''}} គឺជាប្រវែងសរុបនៃគែមទាំងអស់នោះត្រូវគ្នាទៅនឹងកំពូលនៃប្រអប់នោះ។ យើងមាន {{math|2<sup>''n''</sup>}} កំពូល ដូចនេះយើងត្រូវគុណនឹង&nbsp;{{math|2<sup>''n''</sup>}}; ព្រោះថាគែមនីមួយៗត្រូវប៉ះទៅនឹងកំពូលពីរ ជានិច្ច ដែលធ្វើឲ្យគ្រប់គែមទាំងអស់ត្រូវបានរាប់ចំនួនពីរដង។ ដូច្នេះយើងចែកនឹង&nbsp;{{math|2}} ហើយសន្មត់ថាយើងមាន {{math|2<sup>''n''−1</sup>''n''}} ជ្រុង។ ដោយសារតែចាមានគែមជាច្រើនដែលមានប្រវែងស្មើៗគ្នា និង {{mvar|n}} ជ្រុង; ដូចនេះយើងមាន {{math|2<sup>''n''−1</sup>}} គែមនៃប្រវែងនីមួយៗនិងប្រវែងសរុបនៃប្រវែងរបស់គែមទាំងអស់នោះគឺ {{math|2<sup>''n''−1</sup>(''x''<sub>1</sub> + ''x''<sub>2</sub> + · · · + ''x<sub>n</sub>'')}}។ ម្យ៉ាងទៀតៈ
 
:<math>2^{n-1} n \sqrt[n]{x_1 x_2 \cdots x_n}</math>
 
គឺជាប្រវែងសរុបនៃគែមដែឡភ្ជាប់ទៅនឹងកំពូលមួយនៅលើគូបនៅក្នុង {{mvar|n}}-វិមាត្រដែលមានមាឌប៉ុនគ្នា។​​ យោងទៅតាមវិសមភាពដែលបានចែងថាៈ
 
:<math>{x_1 + x_2 +\cdots + x_n \over n} \ge \sqrt[n]{x_1 x_2\cdots x_n}, </math>
 
យើងទទួលបាន
 
:<math>2^{n-1}(x_1 + x_2 + \cdots + x_n) \ge 2^{n-1} n \sqrt[n]{x_1 x_2\cdots x_n}\,</math>
 
ដែលសមភាពកើតឡើងលុះត្រាតែៈ {{math|''x''<sub>1</sub> {{=}} ''x''<sub>2</sub> {{=}} · · · {{=}} ''x<sub>n</sub>''}}.
 
ដូច្នេះ វិសមភាព AM–GM បញ្ជាក់ថាមានតែ [[Hypercube|{{mvar|n}}-គូប]] ប៉ុណ្ណោះដែលមានផលបូកនៃប្រវែងសរុបរបស់គែមដែលភ្ជាប់ទៅនឹងកំពូលនីមួយៗ ទាំងអស់មានតម្លៃតូចជាងគេ ក្នុងចំនោមប្រអប់ដែលមានមាឌ ប៉ុនគ្នាទាំងអស់នៅក្នុង {{mvar|n}}-វិមាត្រ។<ref>{{cite book
| last = Steele
| first = J. Michael
| title = The Cauchy-Schwarz Master Class: An Introduction to the Art of Mathematical Inequalities
| publisher = Cambridge University Press
| series = MAA Problem Books Series
| year = 2004
| isbn = 978-0-521-54677-5
| oclc = 54079548
}}</ref>
 
==ឩទាហរណ៍អំពីការប្រើប្រាស់==
 
ឩបមាថាយើងមានអនុគមន៍ដូចខាងក្រោមៈ
 
:<math>f(x,y,z) = \frac{x}{y} + \sqrt{\frac{y}{z}} + \sqrt[3]{\frac{z}{x}}</math>
 
ចំពោះគ្រប់ចំនួនពិតវិជ្ជមាន {{mvar|x}}, {{mvar|y}} និង&nbsp;{{mvar|z}}. ហើយយើងចង់រកនូវតម្លៃតូចបំផុតនៃ អនុគមន៍មួយនេះ។ ដូចនេះ ជាដំបូងយើងត្រូចសរសេរវាសារជាថ្មីឲ្យទៅជារាងដូចខាងក្រោមៈ
:<math>
\begin{align}
f(x,y,z)
&= 6 \cdot \frac{
\frac{x}{y} +
\frac{1}{2} \sqrt{\frac{y}{z}} + \frac{1}{2} \sqrt{\frac{y}{z}} +
\frac{1}{3} \sqrt[3]{\frac{z}{x}} +
\frac{1}{3} \sqrt[3]{\frac{z}{x}} +
\frac{1}{3} \sqrt[3]{\frac{z}{x}} }
{6} &= 6 \cdot \frac{x_1+x_2+x_3+x_4+x_5+x_6}{6}
\end{align}</math>
ដែល
:<math> x_1=\frac{x}{y},\qquad x_2=x_3=\frac{1}{2} \sqrt{\frac{y}{z}},\qquad x_4=x_5=x_6=\frac{1}{3} \sqrt[3]{\frac{z}{x}}.</math>
 
ដោយប្រើប្រាស់នូវ វិសមភាព AM–GM ចំពោះ {{math|''n'' {{=}} 6}}, យើងបានៈ
 
:<math>
\begin{align}
f(x,y,z)
&\ge 6 \cdot \sqrt[6]{ \frac{x}{y} \cdot \frac{1}{2} \sqrt{\frac{y}{z}} \cdot \frac{1}{2} \sqrt{\frac{y}{z}} \cdot \frac{1}{3} \sqrt[3]{\frac{z}{x}} \cdot \frac{1}{3} \sqrt[3]{\frac{z}{x}} \cdot \frac{1}{3} \sqrt[3]{\frac{z}{x}} }\\
&= 6 \cdot \sqrt[6]{ \frac{1}{2 \cdot 2 \cdot 3 \cdot 3 \cdot 3} \frac{x}{y} \frac{y}{z} \frac{z}{x} }\\
&= 2^{2/3} \cdot 3^{1/2}.
\end{align}</math>
 
ជាងនេះទៅទៀត, យើងដឹងថាអង្គទាំងពីរនៃវិសមភាពនេះស្មើគ្នាទៅបានលុះត្រាតែ គ្រប់តួទាំងអស់មានតម្លៃស្មើគ្នា គឺៈ
 
:<math>f(x,y,z) = 2^{2/3} \cdot 3^{1/2} </math> នៅពេល <math> \frac{x}{y} = \frac{1}{2} \sqrt{\frac{y}{z}} = \frac{1}{3} \sqrt[3]{\frac{z}{x}}</math> ។
 
គ្រប់ចំនុច {{math|(''x'', ''y'', ''z'')}} ដែលផ្ទៀងផ្ទាត់នូវលក្ខ័ណ្ឌនេះសុទ្ធតែ ស្ថិតនៅលើកន្លះបន្ទាត់កាត់តាមគល់តម្រុយ និងមាន​សមីការៈ
 
:<math>(x,y,z)=\biggr(x,\sqrt[3]{2}\sqrt{3}\,x,\frac{3\sqrt{3}}{2}\,x\biggr)</math> ដែល x > 0 ។
 
==ការអនុវត្តន៍ប្រើប្រាស់==
ការប្រើប្រាស់ដ៏សំខាន់មួយនៃវិសមភាពនេះ គឺនៅក្នុង [[financial mathematics|គណិតវិទ្យាខាងផ្នែកហិរញ្ញវត្ថុ]] គឺត្រូវបានយកទៅប្រើប្រាស់ដើម្បីគណនា [[rate of return|អត្រានៃប្រាក់ចំនេញ]]: [[annualized return|ប្រាក់ចំនេញប្រចាំឆ្នាំ]], ដោយបានគណនាតាមមធ្យមធរណីមាត្រ គឺមានតម្លៃទាបជាងប្រាក់ចំនេញប្រចាំឆ្នាំគិតជាមធ្យមដែលគណនាតាម មធ្យមពិជគណិត (ឬក៏អាចស្មើបើប្រាក់ចំនេញទាំងនោះមានតម្លៃស្មើគ្នា)។ វាសំខាន់ណាស់នៅក្នុងការវិភាគទៅលើ ការធ្វើវិនិយោគទុនដូចនេះ ព្រោះថាប្រាក់ចំនេញជាមធ្យម "as the average return overstates the cumulative effect."
 
==សម្រាយបញ្ជាក់នៃវិសមភាព AM–GM ==
 
យើងមានរបៀបក្នុងការស្រាយបញ្ជាក់វិសមភាព AM–GM នេះ; ឧទាហរណ៍ដូចជា, វាអាចស្រាយបញ្ជាក់ដោយប្រើប្រាស់ [[Jensen's inequality|វិសមភាព Jensen]], ដោយប្រើប្រាស់នូវអនុគមន៍ផត ln({{mvar|x}})។ យើងក៏អាចស្រាយបញ្ជាក់វាតាម [[rearrangement inequality|វិសមភាពតំរៀប]]។ ផ្តោតទៅលើប្រវែងនិងតម្រូវការ, ការស្រាយបញ្ជាក់តាមកំនើនវាចារ ខាងក្រោមប្រហែលជា ជាការស្រាយបញ្ជាក់ដ៍ល្អមយួៈ
 
===គំនិតនៃការស្រាយបញ្ជាក់ដំបូងទាំងពីរ===
 
យើងត្រូវតែស្រាយបញ្ជាក់ថាៈ
 
:<math>\frac{x_1+x_2+\cdots+x_n}{n} \ge \sqrt[n]{x_1x_2 \cdots x_n}</math>
 
ដែលសមភាពកើតឡើងលុះត្រាតែគ្រប់ចំនួនទាំងអស់ស្មើគ្នា។ បើ {{math|''x<sub>i</sup>'' ≠ ''x<sub>j</sup>''}}, យើងជំនួស {{mvar|x<sub>i</sup>}} និង {{mvar|x<sub>j</sup>}} ដោយ {{math|(''x<sub>i</sup>'' + ''x<sub>j</sup>'')/2}} នឹងធ្វើឲ្យមធ្យមនព្វន្តដែលនៅខាងឆ្វេងដៃ មិនមានការប្រែប្រួល តែវានឹងធ្វើឲ្យមធ្យមធរណីមាត្រដែលនៅខាងស្ដាំដៃមានការកើនឡើង ព្រោះ ៖
 
:<math>\Bigl(\frac{x_i+x_j}{2}\Bigr)^2-x_ix_j=\Bigl(\frac{x_i-x_j}{2}\Bigr)^2>0 .</math>
 
ដូចនេះអង្គខាងស្ដាំនឹង ធំជាងគេ — ដូចនេះគំនិតគឺថា — នៅពេលដែលគ្រប់ {{mvar|x<sub>i</sup>}} ទាំងអស់មានតម្លៃស្មើទៅនឹងមធ្យមនព្វន្ត ៖
 
:<math>\alpha=\frac{x_1+x_2+\ldots+x_n}{n},</math>
 
ដូចនេះ ខាងក្រោមនេះគឺជាតម្លៃធំជាងគេនៃអង្គខាងស្ដាំ គឺ៖
 
:<math>\frac{x_1+x_2+\ldots+x_n}{n}=\alpha=\sqrt[n]{\alpha\alpha \ldots \alpha}\ge\sqrt[n]{x_1x_2 \ldots x_n}.</math>
 
ការស្រាយបញ្ជាក់នេះគឺពិតចំពោះករណី {{math|''n'' {{=}} 2}} តែដំណើរការក្នុងការយកមធ្យមនៃគូរចំនួនដែលស្រដៀងគ្នា អាចឈានទៅរកភាពបរាជ័យក្នុងការបង្កើតនូវ {{mvar|n}} ចំនួនដែលស្មើគ្នា ក្នុងករណី {{math|''n'' ≥ 3}}។ ជាឧទាហរណ៍មួយក្នុងករណីនេះ គឺ {{math|''x''<sub>1</sub> {{=}} ''x''<sub>2</sub> ≠ ''x''<sub>3</sub>}}: ដោយរកមធ្យមនៃចំនួនពីរដែលខុសគ្នាអាចនឹងបង្កើតបាន ចំនួនពីរដែលមានតម្លៃស្មើគ្នា ប៉ុន្តែ ចំនួនទី បី គឺនៅតែខុសគ្នាដដែល។ ដូចនេះ តាមពិតទៅ យើងមិនដែលទទួលបាននៅ វិសមភាពដែលមាន មធ្យមធរណីមាត្រដែលកើតពី ចំនួន៣ ដែលស្មើគ្នា នោះទេ។
 
ដូច្នេះ ល្បិចមួយទៀតដែលត្រូវប្រើនោះ គឺ ត្រូវតែបង្វែរ គំនិតខាងលើនេះឲ្យទៅជា ការស្រាយបញ្ជាក់ដ៏មានប្រសិទ្ធិ៍ភាពមួយ ចំពោះករណី {{math|''n'' ≥ 3}}។
 
===Hard proof by induction===
 
With the arithmetic mean
:<math>\alpha=\frac{\ x_1 + \cdots + x_n}n</math>
of the non-negative real numbers {{math|''x''<sub>1</sub>, . . . , ''x<sub>n</sub>''}}, the AM–GM statement is equivalent to
:<math>\alpha^n\ge x_1 x_2 \cdots x_n\,</math>
with equality if and only if {{math|''α'' {{=}} ''x<sub>i</sup>''}} for all {{math|''i'' ∈ {1, . . . , ''n''}}}.
 
For the following proof we apply [[mathematical induction]] and only well-known rules of arithmetic.
 
'''Induction basis:''' For {{math|''n'' {{=}} 1}} the statement is true with equality.
 
'''Induction hypothesis:''' Suppose that the AM–GM statement holds for all choices of {{mvar|n}} non-negative real numbers.
 
'''Induction step:''' Consider {{math|''n'' + 1}} non-negative real numbers. Their arithmetic mean&nbsp;{{mvar|α}} satisfies
:<math> (n+1)\alpha=\ x_1 + \cdots + x_n + x_{n+1}.\,</math>
If all numbers are equal to&nbsp;{{mvar|α}}, then we have equality in the AM–GM statement and we are done. Otherwise we may find one number that is greater than&nbsp;{{mvar|α}} and one that is smaller than&nbsp;{{mvar|α}}, say {{math|''x<sub>n</sup>'' > ''α''}} and {{math|''x''<sub>''n''+1</sup> < ''α''}}. Then
 
:<math>(x_n-\alpha)(\alpha-x_{n+1})>0\,.\qquad(*)</math>
 
Now consider the {{mvar|n}} numbers {{math|''x''<sub>1</sub>, . . . , ''x''<sub>''n''–1</sub>, ''y''}} with
:<math>y:=x_n+x_{n+1}-\alpha\ge x_n-\alpha>0\,,</math>
 
which are also non-negative. Since
 
:<math>n\alpha=x_1 + \cdots + x_{n-1} + \underbrace{x_n+x_{n+1}-\alpha}_{=\,y},</math>
 
{{mvar|α}} is also the arithmetic mean of {{mvar|n}} numbers {{math|''x''<sub>1</sub>, . . . , ''x''<sub>''n''–1</sub>, ''y''}} and the induction hypothesis implies
 
:<math>\alpha^{n+1}=\alpha^n\cdot\alpha\ge x_1x_2 \cdots x_{n-1} y\cdot\alpha.\qquad(**)</math>
 
Due to (*) we know that
 
:<math>(\underbrace{x_n+x_{n+1}-\alpha}_{=\,y})\alpha-x_nx_{n+1}=(x_n-\alpha)(\alpha-x_{n+1})>0,</math>
 
hence
 
:<math>y\alpha>x_nx_{n+1}\,,\qquad({*}{*}{*})</math>
 
in particular {{math|''α'' > 0}}. Therefore, if at least one of the numbers {{math|''x''<sub>1</sub>, . . . , ''x''<sub>''n''–1</sub>}} is zero, then we already have strict inequality in (**). Otherwise the right-hand side of (**) is positive and strict inequality is obtained by using the estimate (***) to get a lower bound of the right-hand side of (**). Thus, in both cases we get
 
:<math>\alpha^{n+1}>x_1x_2 \cdots x_{n-1} x_nx_{n+1}\,,</math>
 
which completes the proof.
 
=== Proof by Cauchy using forward-backward-induction===
 
The following proof by cases relies directly on well-known rules of arithmetic but employs the rarely used technique of forward-backward-induction. It is essentially from [[Augustin Louis Cauchy]] and can be found in his ''[[Augustin_Louis_Cauchy#Cours_d.27Analyse|Cours d'analyse]]''.<ref>Cauchy, Augustin-Louis (1821). [http://visualiseur.bnf.fr/Visualiseur?Destination=Gallica&O=NUMM-29058 ''Cours d'analyse de l'École Royale Polytechnique, première partie, Analyse algébrique,''] Paris. The proof of the inequality of arithmetic and geometric means can be found on pages 457ff.</ref>
 
==== The case where all the terms are equal ====
 
If all the terms are equal:
 
<math>x_1 = x_2 = \cdots = x_n,</math>
 
then their sum is {{math|''nx''<sub>1</sub>}}, so their arithmetic mean is&nbsp;{{math|''x''<sub>1</sub>}}; and their product is {{math|''x''<sub>1</sub><sup>''n''</sup>}}, so their geometric mean is&nbsp;{{math|''x''<sub>1</sub>}}; therefore, the arithmetic mean and geometric mean are equal, as desired.
 
==== The case where not all the terms are equal ====
 
It remains to show that if ''not'' all the terms are equal, then the arithmetic mean is greater than the geometric mean. Clearly, this is only possible when {{math|''n'' > 1}}.
 
This case is significantly more complex, and we divide it into subcases.
 
===== The subcase where ''n'' <nowiki>=</nowiki> 2 =====
 
If {{math|''n'' {{=}} 2}}, then we have two terms, {{math|''x''<sub>1</sub>}} and {{math|''x''<sub>2</sub>}}, and since (by our assumption) not all terms are equal, we have:
 
:<math>\begin{align}
\Bigl(\frac{x_1+x_2}{2}\Bigr)^2-x_1x_2
&=\frac14(x_1^2+2x_1x_2+x_2^2)-x_1x_2\\
&=\frac14(x_1^2-2x_1x_2+x_2^2)\\
&=\Bigl(\frac{x_1-x_2}{2}\Bigr)^2>0,
\end{align} </math>
 
hence
 
:<math>
\frac{x_1 + x_2}{2} > \sqrt{x_1 x_2}</math>
 
as desired.
 
===== The subcase where ''n'' <nowiki>=</nowiki> 2<sup>''k''</sup> =====
 
Consider the case where {{math|''n'' {{=}} 2<sup>''k''</sup>}}, where {{mvar|k}} is a positive integer. We proceed by mathematical induction.
 
In the base case, {{math|''k'' {{=}} 1}}, so {{math|''n'' {{=}} 2}}. We have already shown that the inequality holds when {{math|''n'' {{=}} 2}}, so we are done.
 
Now, suppose that for a given {{math|''k'' > 1}}, we have already shown that the inequality holds for {{math|''n'' {{=}} 2<sup>''k''−1</sup>}}, and we wish to show that it holds for {{math|''n'' {{=}} 2<sup>''k''</sup>}}. To do so, we apply the inequality twice for {{math|2<sup>''k''-1</sup>}} numbers and once for {{math|2}} numbers to obtain:
 
: <math>
\begin{align}
\frac{x_1 + x_2 + \cdots + x_{2^k}}{2^k} & {} =\frac{\frac{x_1 + x_2 + \cdots + x_{2^{k-1}}}{2^{k-1}} + \frac{x_{2^{k-1} + 1} + x_{2^{k-1} + 2} + \cdots + x_{2^k}}{2^{k-1}}}{2} \\[7pt]
& \ge \frac{\sqrt[2^{k-1}]{x_1 x_2 \cdots x_{2^{k-1}}} + \sqrt[2^{k-1}]{x_{2^{k-1} + 1} x_{2^{k-1} + 2} \cdots x_{2^k}}}{2} \\[7pt]
& \ge \sqrt{\sqrt[2^{k-1}]{x_1 x_2 \cdots x_{2^{k-1}}} \sqrt[2^{k-1}]{x_{2^{k-1} + 1} x_{2^{k-1} + 2} \cdots x_{2^k}}} \\[7pt]
& = \sqrt[2^k]{x_1 x_2 \cdots x_{2^k}}
\end{align}
</math>
 
where in the first inequality, the two sides are equal only if
 
:<math>x_1 = x_2 = \cdots = x_{2^{k-1}}</math>
 
and
 
:<math>x_{2^{k-1}+1} = x_{2^{k-1}+2} = \cdots = x_{2^k}</math>
 
(in which case the first arithmetic mean and first geometric mean are both equal to&nbsp;{{math|''x''<sub>1</sub>}}, and similarly with the second arithmetic mean and second geometric mean); and in the second inequality, the two sides are only equal if the two geometric means are equal. Since not all {{math|2<sup>''k''</sup>}} numbers are equal, it is not possible for both inequalities to be equalities, so we know that:
 
:<math>\frac{x_1 + x_2 + \cdots + x_{2^k}}{2^k} > \sqrt[2^k]{x_1 x_2 \cdots x_{2^k}}</math>
 
as desired.
 
===== The subcase where ''n'' < 2<sup>''k''</sup> =====
 
If {{mvar|n}} is not a natural power of&nbsp;{{math|2}}, then it is certainly ''less'' than some natural power of 2, since the sequence {{math|2, 4, 8, . . . , 2<sup>''k''</sup>, . . .}} is unbounded above. Therefore, without loss of generality, let {{mvar|m}} be some natural power of {{math|2}} that is greater than&nbsp;{{mvar|n}}.
 
So, if we have {{mvar|n}} terms, then let us denote their arithmetic mean by&nbsp;{{mvar|α}}, and expand our list of terms thus:
 
:<math>x_{n+1} = x_{n+2} = \cdots = x_m = \alpha.</math>
 
We then have:
 
: <math>
\begin{align}
\alpha & = \frac{x_1 + x_2 + \cdots + x_n}{n} \\[6pt]
& = \frac{\frac{m}{n} \left( x_1 + x_2 + \cdots + x_n \right)}{m} \\[6pt]
& = \frac{x_1 + x_2 + \cdots + x_n + \frac{m-n}{n} \left( x_1 + x_2 + \cdots + x_n \right)}{m} \\[6pt]
& = \frac{x_1 + x_2 + \cdots + x_n + \left( m-n \right) \alpha}{m} \\[6pt]
& = \frac{x_1 + x_2 + \cdots + x_n + x_{n+1} + \cdots + x_m}{m} \\[6pt]
& > \sqrt[m]{x_1 x_2 \cdots x_n x_{n+1} \cdots x_m} \\[6pt]
& = \sqrt[m]{x_1 x_2 \cdots x_n \alpha^{m-n}}\,,
\end{align}
</math>
 
so
 
:<math>\alpha^m > x_1 x_2 \cdots x_n \alpha^{m-n}</math>
 
and
 
:<math>\alpha > \sqrt[n]{x_1 x_2 \cdots x_n}</math>
 
as desired.
 
===Proof by induction using basic calculus===
 
The following proof uses mathematical induction and some basic [[differential calculus]].
 
'''Induction basis''': For {{math|''n'' {{=}} 1}} the statement is true with equality.
 
'''Induction hypothesis''': Suppose that the AM–GM statement holds for all choices of {{mvar|n}} non-negative real numbers.
 
'''Induction step''': In order to prove the statement for {{math|''n'' + 1}} non-negative real numbers {{math|''x''<sub>1</sub>, . . . , ''x<sub>n</sub>'', ''x''<sub>''n''+1</sub>}}, we need to prove that
 
:<math>\frac{x_1 + \cdots + x_n + x_{n+1}}{n+1} - ({x_1 \cdots x_n x_{n+1}})^{\frac{1}{n+1}}\ge0</math>
 
with equality only if all the {{math|''n'' + 1}} numbers are equal.
 
If all numbers are zero, the inequality holds with equality. If some but not all numbers are zero, we have strict inequality. Therefore, we may assume in the following, that all {{math|''n'' + 1}} numbers are positive.
 
We consider the last number {{math|''x''<sub>''n''+1</sub>}} as a variable and define the function
:<math> f(t)=\frac{x_1 + \cdots + x_n + t}{n+1} - ({x_1 \cdots x_n t})^{\frac{1}{n+1}},\qquad t>0.</math>
 
Proving the induction step is equivalent to showing that {{math|''f''(''t'') ≥ 0}} for all {{math|''t'' > 0}}, with {{math|''f''(''t'') {{=}} 0}} only if {{math|''x''<sub>1</sub>, . . . , ''x<sub>n</sub>''}} and&nbsp;{{mvar|t}} are all equal. This can be done by analyzing the [[critical point (mathematics)|critical points]] of&nbsp;{{mvar|f}} using some basic calculus.
 
The first [[derivative]] of {{mvar|f}} is given by
 
:<math>f'(t)=\frac{1}{n+1}-\frac{1}{n+1}({x_1 \cdots x_n})^{\frac{1}{n+1}}t^{-\frac{n}{n+1}},\qquad t>0.</math>
 
A critical point {{math|''t''<sub>0</sub>}} has to satisfy {{math|''f′''(''t''<sub>0</sub>) {{=}} 0}}, which means
 
:<math>({x_1 \cdots x_n})^{\frac{1}{n+1}}t_0^{-\frac{n}{n+1}}=1.</math>
 
After a small rearrangement we get
:<math>t_0^{\frac{n}{n+1}}=({x_1 \cdots x_n})^{\frac{1}{n+1}},</math>
 
and finally
 
:<math>t_0=({x_1 \cdots x_n})^{\frac{1}n},</math>
 
which is the geometric mean of {{math|''x''<sub>1</sub>, . . . , ''x<sub>n</sub>''}}. This is the only critical point of&nbsp;{{mvar|f}}. Since {{math|''f′′''(''t'') > 0}} for all {{math|''t'' > 0}}, the function&nbsp;{{mvar|f}} is [[strictly convex function|strictly convex]] and has a strict [[global minimum]] at&nbsp;{{math|''t''<sub>0</sub>}}. Next we compute the value of the function at this global minimum:
 
:<math>
\begin{align}
f(t_0) &= \frac{x_1 + \cdots + x_n + ({x_1 \cdots x_n})^{1/n}}{n+1} - ({x_1 \cdots x_n})^{\frac{1}{n+1}}({x_1 \cdots x_n})^{\frac{1}{n(n+1)}}\\
&= \frac{x_1 + \cdots + x_n}{n+1} + \frac{1}{n+1}({x_1 \cdots x_n})^{\frac{1}n} - ({x_1 \cdots x_n})^{\frac{1}n}\\
&= \frac{x_1 + \cdots + x_n}{n+1} - \frac{n}{n+1}({x_1 \cdots x_n})^{\frac{1}n}\\
&= \frac{n}{n+1}\Bigl(\frac{x_1 + \cdots + x_n}n - ({x_1 \cdots x_n})^{\frac{1}n}\Bigr)\ge0,
\end{align}.</math>
 
where the final inequality holds due to the induction hypothesis. The hypothesis also says that we can have equality only when {{math|''x''<sub>1</sub>, . . . , ''x<sub>n</sub>''}} are all equal. In this case, their geometric mean &nbsp;{{math|''t''<sub>0</sub>}} has the same value, Hence, unless {{math|''x''<sub>1</sub>, . . . , ''x<sub>n</sub>'', ''x''<sub>''n''+1</sub>}} are all equal, we have {{math|''f''(''x''<sub>''n''+1</sub>) > 0}}. This completes the proof.
 
This technique can be used in the same manner to prove the generalized AM–GM inequality and [[Cauchy–Schwarz inequality]] in Euclidean space {{math|'''R'''<sup>''n''</sup>}}.
 
===Proof by Pólya using the exponential function===
 
[[George Pólya]] provided a proof similar to what follows. Let {{math|''f''(''x'') {{=}} e<sup>''x''–1</sup> – ''x''}} for all real&nbsp;{{mvar|x}}, with first [[derivative (mathematics)|derivative]] {{math|''f′''(''x'') {{=}} e<sup>''x''–1</sup> – 1}} and second derivative {{math|''f′′''(''x'') {{=}} e<sup>''x''–1</sup>}}. Observe that {{math|''f''(1) {{=}} 0}}, {{math|''f′''(1) {{=}} 0}} and {{math|''f′′''(''x'') > 0}} for all real&nbsp;{{mvar|x}}, hence {{mvar|f}} is strictly convex with the absolute minimum at {{math|''x'' {{=}} 1}}. Hence {{math|''x'' ≤ e<sup>''x''–1</sup>}} for all real&nbsp;{{mvar|x}} with equality only for {{math|''x'' {{=}} 1}}.
 
Consider a list of non-negative real numbers {{math|''x''<sub>1</sub>, ''x''<sub>2</sub>, . . . , ''x<sub>n</sub>''}}. If they are all zero, then the AM–GM inequality holds with equality. Hence we may assume in the following for their arithmetic mean {{math|''α'' > 0}}. By {{mvar|n}}-fold application of the above inequality, we obtain that
 
:<math>\begin{align}{ \frac{x_1}{\alpha} \frac{x_2}{\alpha} \cdots \frac{x_n}{\alpha} } &\le { e^{\frac{x_1}{\alpha} - 1} e^{\frac{x_2}{\alpha} - 1} \cdots e^{\frac{x_n}{\alpha} - 1} }\\
& = \exp \Bigl( \frac{x_1}{\alpha} - 1 + \frac{x_2}{\alpha} - 1 + \cdots + \frac{x_n}{\alpha} - 1 \Bigr), \qquad (*)
\end{align}</math>
 
with equality if and only if {{math|''x<sub>i</sub>'' {{=}} ''α''}} for every {{math|''i'' ∈ <nowiki>{</nowiki>1, . . . , ''n''<nowiki>}</nowiki>}}. The argument of the exponential function can be simplified:
 
:<math>\begin{align}
\frac{x_1}{\alpha} - 1 + \frac{x_2}{\alpha} - 1 + \cdots + \frac{x_n}{\alpha} - 1 & = \frac{x_1 + x_2 + \cdots + x_n}{\alpha} - n \\
& = n - n \\
& = 0.
\end{align}</math>
 
Returning to {{math|(*)}},
 
:<math>\frac{x_1 x_2 \cdots x_n}{\alpha^n} \le e^0 = 1,</math>
 
which produces {{math|''x''<sub>1</sub> ''x''<sub>1</sub> · · · ''x<sub>n</sub>'' ≤ ''α<sup>n</sup>''}}, hence the result<ref>{{cite book
| last1 = Arnold
| first1 = Denise
| last2 = Arnold
| first2 = Graham
| title = Four unit mathematics
| publisher = Hodder Arnold H&S
| year = 1993
| isbn = 978-0-340-54335-1
| oclc = 38328013
| page = 242
}}</ref>
:<math>\sqrt[n]{x_1 x_2 \cdots x_n} \le \alpha.</math>
 
==Generalizations==
 
===Weighted AM–GM inequality===
 
There is a similar inequality for the [[weighted arithmetic mean]] and [[weighted geometric mean]]. Specifically, let the nonnegative numbers
{{math|''x''<sub>1</sub>, ''x''<sub>2</sub>, . . . , ''x<sub>n</sub>''}} and the nonnegative weights {{math|''w''<sub>1</sub>, ''w''<sub>2</sub>, . . . , ''w<sub>n</sub>''}} be given. Set {{math|''w'' {{=}} ''w''<sub>1</sub> + ''w''<sub>2</sub> + · · · + ''w<sub>n</sub>''}}. If&nbsp;{{math|''w'' > 0}}, then the inequality
 
: <math>\frac{w_1 x_1 + w_2 x_2 + \cdots + w_n x_n}{w} \ge \sqrt[w]{x_1^{w_1} x_2^{w_2} \cdots x_n^{w_n}}</math>
 
holds with equality if and only if all the {{mvar|x<sub>k</sub>}} with {{math|''w<sub>k</sub>'' > 0}} are equal. Here the convention {{math|0<sup>0</sup> {{=}} 1}} is used.
 
If all {{math|''w<sub>k</sub>'' {{=}} 1}}, this reduces to the above inequality of arithmetic and geometric means.
 
===Proof using Jensen's inequality===
 
Using the finite form of [[Jensen's inequality]] for the [[natural logarithm]], we can prove the inequality between the weighted arithmetic mean and the weighted geometric mean stated above.
 
Since an {{mvar|x<sub>k</sub>}} with weight {{math|''w<sub>k</sub>'' {{=}} 0}} has no influence on the inequality, we may assume in the following that all weights are positive. If all {{mvar|x<sub>k</sub>}} are equal, then equality holds. Therefore, it remains to prove strict inequality if they are not all equal, which we will assume in the following, too. If at least one {{mvar|x<sub>k</sub>}} is zero (but not all), then the weighted geometric mean is zero, while the weighted arithmetic mean is positive, hence strict inequality holds. Therefore, we may assume also that all {{mvar|x<sub>k</sub>}} are positive.
 
Since the natural logarithm is [[concave function|strictly concave]], the finite form of Jensen's inequality and the [[functional equation]]s of the natural logarithm imply
:<math>\begin{align}
\ln\Bigl(\frac{w_1x_1+\cdots+w_nx_n}w\Bigr) & >\frac{w_1}w\ln x_1+\cdots+\frac{w_n}w\ln x_n \\
& =\ln \sqrt[w]{x_1^{w_1} x_2^{w_2} \cdots x_n^{w_n}}.
\end{align}</math>
 
Since the natural logarithm is [[monotonic function|strictly increasing]],
:<math>
\frac{w_1x_1+\cdots+w_nx_n}w
>\sqrt[w]{x_1^{w_1} x_2^{w_2} \cdots x_n^{w_n}}.
</math>
 
===Other generalizations===
Other generalizations of the inequality of arithmetic and geometric means include:
 
* [[Muirhead's inequality]],
* [[Maclaurin's inequality]],
* [[Generalized mean|Generalized mean inequality]].
 
==See also==
*[[Ky Fan inequality]]
*[[Young's inequality]]
 
==Notes==
{{reflist|group=note}}
 
==References==
{{Reflist}}
 
==External links==
* {{cite web|title=Introduction to Inequalities|url=http://www.mediafire.com/?1mw1tkgozzu |author=Arthur Lohwater|year=1982|publisher=Online e-book in PDF format}}
 
{{DEFAULTSORT:Inequality Of Arithmetic And Geometric Means}}
[[:Category:Inequalities]]
[[:Category:Means]]
[[:Category:Articles containing proofs]]
[[:Category:ភាសាខ្មែរ]]